Skip to main content
Log in

Incomplete contracts with disparity, uncertainty, information and incentives

  • Published:
Theory and Decision Aims and scope Submit manuscript

Abstract

Cooperation among firms is governed by contracts. An interesting phenomenon is that some contracts are comprehensive while some are limited. This study tries to explain different levels of the incompleteness of contracts that firms choose to govern their cooperation with. We find that a limited contract is more efficient than a comprehensive contract if partners are highly disparate or product quality is largely uncertain, and vice versa. In contrast, if there are private information and incentives to invest in quality, a comprehensive contract is likely to be more efficient. These findings offer an understanding as to why incomplete contracts are so popular in practice.

This is a preview of subscription content, log in via an institution to check access.

Access this article

Price excludes VAT (USA)
Tax calculation will be finalised during checkout.

Instant access to the full article PDF.

Fig. 1
Fig. 2
Fig. 3
Fig. 4
Fig. 5
Fig. 6
Fig. 7
Fig. 8
Fig. 9

Similar content being viewed by others

Notes

  1. The firms’ knowledge about quality θ differ in Model 2, and they can take action to improve quality θ in Model 3. We use one parameter θ for convenience; extending to many parameters is straightforward.

  2. For convenience, we use f(·) to denote all density functions; the same treatment for revenue function R(·).

  3. Although contractual items such as si(x) cannot be dependent on θ, non-contractual items can still be dependent on θ if θ is known at the time.

  4. There are two theoretical approaches to contracting: the principal-agent setting, and the bargaining setting. We use the bargaining setting. These two settings are equivalent to a large extent; see Wang (2012, p. 30) for an explanation.

  5. For example, for a Cobb–Douglas production function \(y={x}_{1}^{\alpha }{x}_{2}^{1-\alpha }\), there is a parameter α representing some inherent characteristics, and x1 and x2 are action variables. However, with endogenous growth, α can be dependent on actions.

  6. An easy way to understand this derivative is to assume that the density function is discrete.

References

  • Aghion, P., & Bolton, P. (1992). An incomplete contracts approach to financial contracting. Review of Economic Studies, 59, 473–494.

    Article  Google Scholar 

  • Dye, R. A. (1985). Costly contract contingencies. International Economic Review, 26(1), 233–250.

    Article  MathSciNet  Google Scholar 

  • Grossman, S. J., & Hart, O. D. (1986). The costs and benefits of ownership: A theory of vertical and lateral integration. Journal of Political Economy, 94(4), 691–719.

    Article  Google Scholar 

  • Hart, O. D. (1988). Incomplete contracts and the theory of the firm. Journal of Law, Economics, and Organization, 4(1), 119–139.

    Google Scholar 

  • Hart, O. D. (1995). Firms, contracts, and financial structure. Clarendon Press.

    Book  Google Scholar 

  • Hart, O. D., & Moore, J. (1988). Incomplete contracts and renegotiation. Econometrica, 56, 755–786.

    Article  MathSciNet  Google Scholar 

  • Hart, O. D., & Moore, J. (1990). Property rights and the nature of the firm. Journal of Political Economy, 98(6), 1119–1158.

    Article  Google Scholar 

  • Kaplan, S. N., & Strömberg, P. (2003). Financial contracting theory meets the real world: An empirical analysis of venture capital contracts. Review of Economic Studies, 70, 281–315.

    Article  Google Scholar 

  • Maskin, E. (2002). On indescribable contingencies and incomplete contracts. European Economic Review, 46, 725–733.

    Article  Google Scholar 

  • Maskin, E., & Tirole, J. (1999a). Unforeseen contingencies and incomplete contracts. Review of Economic Studies, 66, 83–114.

    Article  MathSciNet  Google Scholar 

  • Maskin, E., & Tirole, J. (1999b). Two remarks on the property-rights literature. Review of Economic Studies, 66, 139–149.

    Article  Google Scholar 

  • Schmitz, P. W. (2006). Information gathering, transaction costs, and the property rights approach. American Economic Review, 96(1), 422–434.

    Article  Google Scholar 

  • Scott, R. (2003). A theory of self-enforcing indefinite agreements. Columbia Law Review, 102, 1641–1699.

    Article  Google Scholar 

  • Segal, I. (1999). Complexity and renegotiation: A foundation for incomplete contracts. Review of Economic Studies, 66, 57–82.

    Article  MathSciNet  Google Scholar 

  • Tirole, J. (1999). Incomplete contracts: Where do we stand? Econometrica, 67(4), 741–781.

    Article  Google Scholar 

  • Wang, S. (2010). Contractual Incompleteness for External Risks. B.E. Journal of Theoretical Economics, 10(1), Article 19.

    Article  MathSciNet  Google Scholar 

  • Wang, S. (2012). Organization theory and its applications. Routledge Publisher.

    Book  Google Scholar 

Download references

Acknowledgements

We would like to thank an anonymous referee whose comments and suggestions improve this paper considerably.

Author information

Authors and Affiliations

Authors

Corresponding author

Correspondence to Susheng Wang.

Ethics declarations

Conflict of interest

More specifically, 1. The author declares that they have no relevant or material financial interests related to the research described in this paper. 2. The author declares that they have no known competing financial interests or personal relationships that could have appeared to influence the work reported in this paper. 3. No data is involved in this research project. 4. No financial support is obtained for this research project. 5. No third party has the right to review this paper prior to its circulation. 6. No IRB approval was sought since this research project does not use human subjects.

Additional information

Publisher's Note

Springer Nature remains neutral with regard to jurisdictional claims in published maps and institutional affiliations.

Appendix

Appendix

1.1 Proof of Lemma 1

Conditions \(FO{C}_{1}\) and \(FO{C}_{2}\) imply

$$\int x{f}_{h}\left(x,\theta ,h\left({a}_{1},{a}_{2}\right)\right)dx=\frac{{c}_{1}^{\prime}\left({a}_{1}\right)}{{h}_{1}^{\prime}\left(a\right)}+\frac{{c}_{2}^{\prime}\left({a}_{2}\right)}{{h}_{2}^{\prime}\left(a\right)},$$

implying

$${R}_{h}^{\mathrm{^{\prime}}}\left(\theta ,h\left({a}_{1},{a}_{2}\right)\right)=\frac{{c}_{1}^{\mathrm{^{\prime}}}\left({a}_{1}\right)}{{h}_{1}^{\mathrm{^{\prime}}}\left(a\right)}+\frac{{c}_{2}^{\mathrm{^{\prime}}}\left({a}_{2}\right)}{{h}_{2}^{\mathrm{^{\prime}}}\left(a\right)}.$$

Hence, problem becomes

$$\begin{array}{l}\underset{{s}_{i}\in \mathcal{S},{\widehat{a}}_{i}\in \mathcal{A}}{{\text{max}}}{E}_{\theta }\left(R\left(\theta ,h\left({\widehat{a}}_{1}\left(\theta \right),{\widehat{a}}_{2}\left(\theta \right)\right)\right)-{c}_{1}\left({\widehat{a}}_{1}\left(\theta \right)\right)-{c}_{2}\left({\widehat{a}}_{2}\left(\theta \right)\right)\right)\\ {\text{ s.t. }} {R}_{h}^{\prime}\left(\theta ,h\left(\widehat{a}\left(\theta \right)\right)\right)=\frac{{c}_{1}^{\mathrm{^{\prime}}}\left({\widehat{a}}_{1}\left(\theta \right)\right)}{{h}_{1}^{\mathrm{^{\prime}}}\left(\widehat{a}\left(\theta \right)\right)}+\frac{{c}_{2}^{\mathrm{^{\prime}}}\left({\widehat{a}}_{2}\left(\theta \right)\right)}{{h}_{2}^{\mathrm{^{\prime}}}\left(\widehat{a}\left(\theta \right)\right)},\\ \quad {FOC}_{2}, \, SO{C}_{1}, \, SO{C}_{2},\\ \quad {s}_{1}\left(x\right)+{s}_{2}\left(x\right)=x,\text{ for all }x\in {\mathbb{R}}_{+}.\end{array}$$

Since the objective function is unrelated to the revenue-sharing scheme, this problem can be solved in two steps. First, we find a solution \(\widehat{a}\left(\theta \right)\) from the following problem:

$$\begin{array}{l}\underset{{\widehat{a}}_{1},{\widehat{a}}_{2}\in \mathcal{A}}{{\text{max}}}{E}_{\theta }\left(R\left(\theta ,h\left({\widehat{a}}_{1}\left(\theta \right),{\widehat{a}}_{2}\left(\theta \right)\right)\right)-{c}_{1}\left({\widehat{a}}_{1}\left(\theta \right)\right)-{c}_{2}\left({\widehat{a}}_{2}\left(\theta \right)\right)\right)\\ \text{ s.t. } {R}_{h}^{\prime}\left(\theta ,h\left(\widehat{a}\left(\theta \right)\right)\right)=\frac{{c}_{1}^{\mathrm{^{\prime}}}\left({\widehat{a}}_{1}\left(\theta \right)\right)}{{h}_{1}^{\mathrm{^{\prime}}}\left(\widehat{a}\left(\theta \right)\right)}+\frac{{c}_{2}^{\mathrm{^{\prime}}}\left({\widehat{a}}_{2}\left(\theta \right)\right)}{{h}_{2}^{\mathrm{^{\prime}}}\left(\widehat{a}\left(\theta \right)\right)}.\end{array}$$

By the Pontryagin maximum principle, this problem can be further changed to the problem. This problem is not related to the revenue-sharing scheme.

Second, given \(\left({\widehat{a}}_{1},{\widehat{a}}_{2}\right)\) from the above, we look for a revenue-sharing scheme \({s}_{i}(x)\) that satisfies \(FO{C}_{2},SO{C}_{1}\) and \(SO{C}_{2}.\) We can first find a sharing scheme \({s}_{2}\left(x\right)\) that satisfies \(FO{C}_{2}\):

$$\int {s}_{2}\left(x\right){f}_{h}\left(x;\theta ,h\left({\widehat{a}}_{1}\left(\theta \right),{\widehat{a}}_{2}\left(\theta \right)\right)\right)dx=\frac{{c}_{2}^{\mathrm{^{\prime}}}\left({\widehat{a}}_{2}\left(\theta \right)\right)}{{h}_{2}^{\mathrm{^{\prime}}}\left(\widehat{a}\left(\theta \right)\right)},$$
(20)

and then define \({s}_{1}\left(x\right)\) by the condition \({s}_{1}\left(x\right)+{s}_{2}\left(x\right)=x\). If \({s}_{2}\left(x\right)\) can be a linear function, as in Case 1, verifying \(SO{C}_{1}\) and \(SO{C}_{2}\) will be easy.

1.2 Proof of Lemma 2

The objective function can be written as

$$U\equiv {r}_{1}{\text{ln}}\left(\int {s}_{1}\left(x\right)f\left(x,h\left(a\right)\right)dx-{c}_{1}\left({a}_{1}\right)\right)+{r}_{2}{\text{ln}}\left(\int {s}_{2}\left(x\right)f\left(x,h\left(a\right)\right)dx-{c}_{2}\left({a}_{2}\right)\right).$$

Then, problem can be written as

$$\begin{array}{l}\underset{{a}_{1},{a}_{2}\in {\mathbb{A}}}{{\text{max}}} \underset{{s}_{i}\in \mathcal{S}}{{\text{max}}} U\\ \text{ s.t. } {s}_{1}\left(x\right)+{s}_{2}\left(x\right)=x,\text{ for all }x\in {\mathbb{R}}_{+}.\end{array}$$
(21)

Given \(\left({a}_{1},{a}_{2}\right)\), consider problem:

$$\begin{array}{l}\underset{{s}_{i}\in \mathcal{S}}{{\text{max}}} U\\ \text{ s.t. } {s}_{1}\left(x\right)+{s}_{2}\left(x\right)=x,\text{ for all }x\in {\mathbb{R}}_{+}.\end{array}$$
(22)

The FOC isFootnote 6

$$\begin{array}{c}0=\frac{\partial U}{\partial {s}_{1}}=\frac{ {r}_{1}f\left(x,h\left(a\right)\right)}{\int {s}_{1}\left(x\right)f\left(x,h\left(a\right)\right)dx-{c}_{1}\left({a}_{1}\right)}-\frac{{r}_{2}f\left(x,h\left(a\right)\right)}{\int \left(x-{s}_{1}\left(x\right)\right)f\left(x,h\left(a\right)\right)dx-{c}_{2}\left({a}_{2}\right)},\end{array}$$

implying

$${r}_{1}\left[R\left(h\left(a\right)\right)-\int {s}_{1}\left(x\right)f\left(x,h\left(a\right)\right)dx-{c}_{2}\left({a}_{2}\right)\right]={r}_{2}\left[\int {s}_{1}\left(x\right)f\left(x,h\left(a\right)\right)dx-{c}_{1}\left({a}_{1}\right)\right],$$

implying

$$\int {s}_{1}\left(x\right)f\left(x,h\left(a\right)\right)dx=\frac{{r}_{1}R\left(h\left(a\right)\right)+{r}_{2}{c}_{1}\left({a}_{1}\right)-{r}_{1}{c}_{2}\left({a}_{2}\right)}{{r}_{1}+{r}_{2}}.$$
(23)

The SOC is satisfied:

$$\frac{{\partial }^{2}U}{\partial {s}_{1}^{2}}=-\frac{ {r}_{1}{\left[f\left(x,h\left(a\right)\right)\right]}^{2}}{{\left[\int {s}_{1}\left(x\right)f\left(x,h\left(a\right)\right)dx-{c}_{1}\left({a}_{1}\right)\right]}^{2}}-\frac{{r}_{2}{\left[f\left(x,h\left(a\right)\right)\right]}^{2}}{{\left[\int \left(x-{s}_{1}\left(x\right)\right)f\left(x,h\left(a\right)\right)dx-{c}_{2}\left({a}_{2}\right)\right]}^{2}}<0.$$

That is, given\(\left({a}_{1},{a}_{2}\right)\), any admissible \({s}_{1}\left(\cdot \right)\) satisfying is optimal. Consider a simple linear sharing scheme\({s}_{1}\left(x\right)={\alpha }_{1}x\). Then, by, we have

$${\alpha }_{1}=\frac{{r}_{1}R\left(h\left(a\right)\right)+{r}_{2}{c}_{1}\left({a}_{1}\right)-{r}_{1}{c}_{2}\left({a}_{2}\right)}{\left({r}_{1}+{r}_{2}\right)R\left(h\left(a\right)\right)}.$$

That is, equation has a simple solution of the form \({s}_{1}\left(x\right)={\alpha }_{1}x\). Then,

$${\alpha }_{2}=1-{\alpha }_{1}=\frac{{r}_{2}R\left(h\left(a\right)\right)-{r}_{2}{c}_{1}\left({a}_{1}\right)+{r}_{1}{c}_{2}\left({a}_{2}\right)}{\left({r}_{1}+{r}_{2}\right)R\left(h\left(a\right)\right)},$$

implying \({s}_{2}\left(x\right)={\alpha }_{2}x.\) Then, the objective function of is

$$\begin{array}{l}{\left({\alpha }_{1}R\left(h\left(a\right)\right)-{c}_{1}\left({a}_{1}\right)\right)}^{{r}_{1}}{\left({\alpha }_{2}R\left(h\left(a\right)\right)-{c}_{2}\left({a}_{2}\right)\right)}^{{r}_{2}}\\ ={\left(\frac{{r}_{1}R\left(h\left(a\right)\right)-{r}_{1}{c}_{1}\left({a}_{1}\right)-{r}_{1}{c}_{2}\left({a}_{2}\right)}{{r}_{1}+{r}_{2}}\right)}^{{r}_{1}}{\left(\frac{{r}_{2}R\left(h\left(a\right)\right)-{r}_{2}{c}_{1}\left({a}_{1}\right)-{r}_{2}{c}_{2}\left({a}_{2}\right)}{{r}_{1}+{r}_{2}}\right)}^{{r}_{2}}\\ =\frac{{r}_{1}^{{r}_{1}}{r}_{2}^{{r}_{2}}}{{\left({r}_{1}+{r}_{2}\right)}^{{r}_{1}+{r}_{2}}}{\left[R\left(h\left(a\right)\right)-{c}_{1}\left({a}_{1}\right)-{c}_{2}\left({a}_{2}\right)\right]}^{{r}_{1}+{r}_{2}}.\end{array}$$

Then, problem becomes

$$\underset{{a}_{1},{a}_{2}\in {\mathbb{A}}}{{\text{max}}} R\left(h\left(a\right)\right)-{c}_{1}\left({a}_{1}\right)-{c}_{2}\left({a}_{2}\right).$$

1.3 Derivation of case 1

We have

$$R\left(\theta ,h\right)=E\left(\widetilde{A}\theta h\right)=\theta h, \, R\left(h\right)={E}_{\theta }\left(R\left(\widetilde{\theta },h\right)\right)=\overline{\theta }h.$$

Then, for a limited contract, problem becomes

$$\begin{array}{l}\underset{ {a}_{1,}{a}_{2}\in A}{\text{max}} \theta h\left({a}_{1},{a}_{2}\right)-\frac{1}{2}{a}_{1}^{2}-\frac{1}{2}{a}_{2}^{2}\\ \text{ s.t. } \theta =\frac{{a}_{1}}{{\mu }_{1}}+\frac{{a}_{2}}{{\mu }_{2}}.\end{array}$$

The solution is

$${a}_{1}=\frac{{\mu }_{1}^{3}}{{\mu }_{1}^{2}+{\mu }_{2}^{2}}\theta , {a}_{2}=\frac{{\mu }_{2}^{3}}{{\mu }_{1}^{2}+{\mu }_{2}^{2}}\theta .$$

Then,

$$\begin{array}{l}{W}_{l}^{S}={E}_{\theta }\left(R\left(\theta ,h\left({a}_{1},{a}_{2}\right)\right)-{c}_{1}\left({a}_{1}\right)-{c}_{2}\left({a}_{2}\right)\right)\\ ={E}_{\theta }\left(\theta \left(\frac{{\mu }_{1}^{4}}{{\mu }_{1}^{2}+{\mu }_{2}^{2}}\theta +\frac{{\mu }_{2}^{4}}{{\mu }_{1}^{2}+{\mu }_{2}^{2}}\theta \right)-\frac{1}{2}{\left(\frac{{\mu }_{1}^{3}}{{\mu }_{1}^{2}+{\mu }_{2}^{2}}\theta \right)}^{2}-\frac{1}{2}{\left(\frac{{\mu }_{2}^{3}}{{\mu }_{1}^{2}+{\mu }_{2}^{2}}\theta \right)}^{2}\right)\\ =\left(\frac{{\mu }_{1}^{4}+{\mu }_{2}^{4}}{{\mu }_{1}^{2}+{\mu }_{2}^{2}}-\frac{{\mu }_{1}^{6}+{\mu }_{2}^{6}}{2{\left({\mu }_{1}^{2}+{\mu }_{2}^{2}\right)}^{2}}\right){E}_{\theta }\left({\theta }^{2}\right) \\ =\frac{{\sigma }_{\theta }^{2}+{\overline{\theta }}^{2}}{2{\left({\mu }_{1}^{2}+{\mu }_{2}^{2}\right)}^{2}}\left({\mu }_{1}^{6}+{\mu }_{2}^{6}+2{\mu }_{1}^{2}{\mu }_{2}^{2}\left({\mu }_{1}^{2}+{\mu }_{2}^{2}\right)\right)\\ =\frac{{\sigma }_{\theta }^{2}+{\overline{\theta }}^{2}}{2}\frac{{\mu }_{1}^{4}+{\mu }_{2}^{4}+{\mu }_{1}^{2}{\mu }_{2}^{2}}{{\mu }_{1}^{2}+{\mu }_{2}^{2}}.\end{array}$$

We also have

$$\frac{{c}_{2}^{\mathrm{^{\prime}}}\left({a}_{2}\right)}{{h}_{2}^{\mathrm{^{\prime}}}\left(a\right)}=\frac{{a}_{2}}{{\mu }_{2}}=\frac{{\mu }_{2}^{2}}{{\mu }_{1}^{2}+{\mu }_{2}^{2}}\theta .$$

Let \({s}_{2}\left(x\right)={\alpha }_{2}x\), where \({\alpha }_{2}\) is a constant. Then,

$$\int {s}_{2}\left(x\right){f}_{h}\left(x,\theta ,h\left({a}_{1},{a}_{2}\right)\right)dx={\alpha }_{2}{R}_{h}^{\mathrm{^{\prime}}}\left(\theta ,h\left(a\right)\right)={\alpha }_{2}\theta .$$

Hence, if\({\alpha }_{2}=\frac{{\mu }_{2}^{2}}{{\mu }_{1}^{2}+{\mu }_{2}^{2}}\), then equation is satisfied for any\(\theta\). That is, there is a linear sharing scheme satisfying equation. We also have

$$h={\mu }_{1}{a}_{1}+{\mu }_{2}{a}_{2}=\frac{{\mu }_{1}^{4}+{\mu }_{2}^{4}}{{\mu }_{1}^{2}+{\mu }_{2}^{2}}\theta .$$

For a comprehensive contract, problem becomes

$${W}_{c}^{S}=\underset{ {a}_{1,}{a}_{2}\in {\mathbb{A}}}{\text{max}} \overline{\theta }h\left({a}_{1},{a}_{2}\right)-\frac{1}{2}{a}_{1}^{2}-\frac{1}{2}{a}_{2}^{2}.$$

The FOCs are

$$\overline{\theta }{\mu }_{1}={a}_{1}, \overline{\theta }{\mu }_{2}={a}_{2}.$$

Then,

$${W}_{c}^{S}=\overline{\theta }\left({\mu }_{1}^{2}\overline{\theta }+{\mu }_{2}^{2}\overline{\theta }\right)-\frac{1}{2}{\left(\overline{\theta }{\mu }_{1}\right)}^{2}-\frac{1}{2}{\left(\overline{\theta }{\mu }_{2}\right)}^{2}=\frac{{\mu }_{1}^{2}+{\mu }_{2}^{2}}{2}{\overline{\theta }}^{2},$$

and

$$h\left(a\right)={\mu }_{1}{a}_{1}+{\mu }_{2}{a}_{2}=\left({\mu }_{1}^{2}+{\mu }_{2}^{2}\right)\overline{\theta },$$

and

$$\begin{array}{l}{\alpha }_{1}=\frac{{r}_{1}\overline{\theta }h\left(a\right)+\frac{{r}_{2}}{2}{a}_{1}^{2}-\frac{{r}_{1}}{2}{a}_{2}^{2}}{({r}_{1}+{r}_{2})\overline{\theta }h\left(a\right)}=\frac{{r}_{1}\left({\mu }_{1}^{2}+{\mu }_{2}^{2}\right){\overline{\theta }}^{2}+\frac{{r}_{2}}{2}{\mu }_{1}^{2}{\overline{\theta }}^{2}-\frac{{r}_{1}}{2}{\mu }_{2}^{2}{\overline{\theta }}^{2}}{\left({r}_{1}+{r}_{2}\right)\left({\mu }_{1}^{2}+{\mu }_{2}^{2}\right){\overline{\theta }}^{2}}=\frac{\left(2{r}_{1}+{r}_{2}\right){\mu }_{1}^{2}+{r}_{1}{\mu }_{2}^{2}}{2\left({r}_{1}+{r}_{2}\right)\left({\mu }_{1}^{2}+{\mu }_{2}^{2}\right)},\\ =\frac{\left({r}_{1}+{r}_{2}\right){\mu }_{1}^{2}+{r}_{1}({\mu }_{1}^{2}+{\mu }_{2}^{2})}{2\left({r}_{1}+{r}_{2}\right)\left({\mu }_{1}^{2}+{\mu }_{2}^{2}\right)}=\frac{1}{2}\left(\frac{{\mu }_{1}^{2}}{{\mu }_{1}^{2}+{\mu }_{2}^{2}}+\frac{{r}_{1}}{{r}_{1}+{r}_{2}}\right),\end{array}$$

and symmetrically,

$${\alpha }_{2}=\frac{1}{2}\left(\frac{{\mu }_{2}^{2}}{{\mu }_{1}^{2}+{\mu }_{2}^{2}}+\frac{{r}_{2}}{{r}_{1}+{r}_{2}}\right).$$

1.4 Proof of Proposition 1

We have \({W}_{l}^{S}>{W}_{c}^{S}\) iff

$$\frac{{\sigma }_{\theta }^{2}+{\overline{\theta }}^{2}}{2}\frac{{\mu }_{1}^{6}-{\mu }_{2}^{6}}{{\mu }_{1}^{4}-{\mu }_{2}^{4}}>\frac{{\mu }_{1}^{2}+{\mu }_{2}^{2}}{2}{\overline{\theta }}^{2},$$

or

$${\left(\frac{{\sigma }_{\theta }}{\overline{\theta }}\right)}^{2}+1>\frac{\left({\mu }_{1}^{2}+{\mu }_{2}^{2}\right)\left({\mu }_{1}^{4}-{\mu }_{2}^{4}\right)}{{\mu }_{1}^{6}-{\mu }_{2}^{6}},$$

or

$${\left(\frac{{\sigma }_{\theta }}{\overline{\theta }}\right)}^{2}>\frac{{\mu }_{1}^{4}{\mu }_{2}^{2}-{\mu }_{1}^{2}{\mu }_{2}^{4}}{{\mu }_{1}^{6}-{\mu }_{2}^{6}}=\frac{{r}_{\mu }^{2}-{r}_{\mu }^{4}}{1-{r}_{\mu }^{6}}=\frac{{r}_{\mu }^{2}}{1+{r}_{\mu }^{2}+{r}_{\mu }^{4}}.$$

The right-hand side of the above inequality is increasing when \({r}_{\mu }<1\) and decreasing when \({r}_{\mu }>1\) and reaches maximum at \({r}_{\mu }=1\).

1.5 Proof of Lemma 3

By \(FO{C}_{1}\) and \(FO{C}_{2}\), we have

$${R}_{h}^{\mathrm{^{\prime}}}\left(\theta ,h\left({a}_{1},{a}_{2}\right)\right)=\frac{{c}_{1}^{\mathrm{^{\prime}}}\left({a}_{1}\right)}{{h}_{1}^{\mathrm{^{\prime}}}\left(a\right)}+\frac{{c}_{2}^{\mathrm{^{\prime}}}\left({a}_{2}\right)}{{h}_{2}^{\mathrm{^{\prime}}}\left(a\right)}.$$

By \(FO{C}_{1}\) and \(FO{C}_{3}\), we have

$$\left({h}_{1}^{\mathrm{^{\prime}}}{a}_{1}^{\mathrm{^{\prime}}}\left(\theta \right)+{h}_{2}^{\mathrm{^{\prime}}}{a}_{2}^{\mathrm{^{\prime}}}\left(\theta \right)\right)\frac{{c}_{1}^{\mathrm{^{\prime}}}\left({a}_{1}\right)}{{h}_{1}^{\mathrm{^{\prime}}}\left(a\right)}={c}_{1}^{\mathrm{^{\prime}}}\left({a}_{1}\left(\theta \right)\right){a}_{1}^{\mathrm{^{\prime}}}\left(\theta \right),$$

implying

$${h}_{1}^{\mathrm{^{\prime}}}{a}_{1}^{\mathrm{^{\prime}}}\left(\theta \right)+{h}_{2}^{\mathrm{^{\prime}}}{a}_{2}^{\mathrm{^{\prime}}}\left(\theta \right)={h}_{1}^{\mathrm{^{\prime}}}\left(a\right){a}_{1}^{\mathrm{^{\prime}}}\left(\theta \right),$$

implying

$${a}_{2}^{\mathrm{^{\prime}}}\left(\theta \right)=0,$$

for all \(\theta\). Then, problem becomes

$$\begin{aligned}&\underset{{s}_{i}\in \mathcal{S},{\widehat{a}}_{i}\in \mathcal{A}}{{\text{max}}}{E}_{\theta }\left(R\left(\theta ,h\left(\widehat{a}\left(\theta \right)\right)\right)-{c}_{1}\left({\widehat{a}}_{1}\left(\theta \right)\right)-{c}_{2}\left({\widehat{a}}_{2}\left(\theta \right)\right)\right)\\ & \quad{\text{s.t}}. {R}_{h}^{\mathrm{^{\prime}}}\left(\theta ,h\left(\widehat{a}\left(\theta \right)\right)\right)=\frac{{c}_{1}^{\mathrm{^{\prime}}}\left({\widehat{a}}_{1}\left(\theta \right)\right)}{{h}_{1}^{\mathrm{^{\prime}}}\left(\widehat{a}\left(\theta \right)\right)}+\frac{{c}_{2}^{\mathrm{^{\prime}}}\left({\widehat{a}}_{2}\left(\theta \right)\right)}{{h}_{2}^{\mathrm{^{\prime}}}\left(\widehat{a}\left(\theta \right)\right)},\\ & \quad {\widehat{a}}_{2}^{\mathrm{^{\prime}}}\left(\theta \right)=0,\\ & \quad {FOC}_{2},SO{C}_{1}, \, SO{C}_{2}, \, SO{C}_{3},\\ & \quad {s}_{1}\left(x\right)+{s}_{2}\left(x\right)=x,\text{ for all }x\in {\mathbb{R}}_{+}.\end{aligned}$$

This problem can be solved in two steps. We first solve for \(\left({a}_{1}\left(\theta \right),{a}_{2}\left(\theta \right)\right)\) from, and then solve for a revenue-sharing scheme \({s}_{i}\left(x\right)\) from the remaining conditions.

1.6 Derivation of Case 2

We have

$$R\left(\theta ,h\right)=E\left(\widetilde{A}\theta h\right)=\theta h, \, R\left(h\right)={E}_{\theta }\left(R\left(\widetilde{\theta },h\right)\right)=\overline{\theta }h.$$

Then, with a limited contract, problem becomes

$$\begin{aligned}&\quad \underset{ {a}_{1,}{a}_{2}\in \mathcal{A}}{\text{max}} {E}_{\theta }\left(\theta h\left({a}_{1}(\theta ),{a}_{2}\left(\theta \right)\right)-\frac{1}{2}{a}_{1}^{2}\left(\theta \right)-\frac{1}{2}{a}_{2}^{2}\left(\theta \right)\right)\\ \text{ s.t. } &\theta =\frac{{a}_{1}\left(\theta \right)}{{\mu }_{1}}+\frac{{a}_{2}\left(\theta \right)}{{\mu }_{2}},\\ & \quad {a}_{2}^{\prime}\left(\theta \right)=0.\end{aligned}$$

By the first condition, we have

$$1=\frac{{a}_{1}^{\mathrm{^{\prime}}}}{{\mu }_{1}}+\frac{{a}_{2}^{\mathrm{^{\prime}}}}{{\mu }_{2}}=\frac{{a}_{1}^{\mathrm{^{\prime}}}}{{\mu }_{1}},$$

implying \({a}_{1}^{\prime}={\mu }_{1}\), implying

$${a}_{1}\left(\theta \right)={\mu }_{1}\theta +{\overline{a} }_{1},$$
(24)

where \({\overline{a} }_{1}\) is a constant. By the second condition, we have

$${a}_{2}\left(\theta \right)={\overline{a} }_{2},$$
(25)

where \({\overline{a} }_{2}\) is a constant. Then, by the first condition in the problem,

$$\theta =\frac{{\mu }_{1}\theta +{\overline{a} }_{1}}{{\mu }_{1}}+\frac{{\overline{a} }_{2}}{{\mu }_{2}},$$

implying

$${\overline{a} }_{2}=-\frac{{\mu }_{2}}{{\mu }_{1}}{\overline{a} }_{1},$$
(26)

implying

$$0=\frac{{\overline{a} }_{1}}{{\mu }_{1}}+\frac{{\overline{a} }_{2}}{{\mu }_{2}}.$$

Since \({a}_{1},{a}_{2}\ge 0\), this implies \({\overline{a} }_{1}={\overline{a} }_{2}=0\). Hence, the solution is

$${a}_{1}\left(\theta \right)={\mu }_{1}\theta , {a}_{2}\left(\theta \right)=0.$$

This implies

$$h={\mu }_{1}^{2}\theta .$$

Then,

$$\begin{array}{c}{W}_{l}^{A}={E}_{\theta }\left(R\left(\theta ,h\left({a}_{1},{a}_{2}\right)\right)-{c}_{1}\left({a}_{1}\right)-{c}_{2}\left({a}_{2}\right)\right)={E}_{\theta }\left(\theta \left({\mu }_{1}^{2}\theta \right)-\frac{1}{2}{\left({\mu }_{1}\theta \right)}^{2}\right)=\frac{1}{2}{\mu }_{1}^{2}{E}_{\theta }\left({\theta }^{2}\right).\end{array}$$

Further, we have

$$\frac{{c}_{2}^{\mathrm{^{\prime}}}\left({a}_{2}\right)}{{h}_{2}^{\mathrm{^{\prime}}}\left(a\right)}=\frac{{a}_{2}}{{\mu }_{2}}=0.$$

Let \({s}_{2}\left(x\right)={s}_{0}+{\alpha }_{2}x,\) where \({s}_{0}\) and \({\alpha }_{2}\) are two constants. Then, equation implies \({\alpha }_{2}=0\) and \({s}_{0}\) can be any value. That is, there is a linear contract satisfying equation.

1.7 Proof of Proposition 2

We have \({W}_{l}^{A}>{W}_{c}^{A}\) iff

$$\frac{1}{2}{\mu }_{1}^{2}\left({\sigma }_{\theta }^{2}+{\overline{\theta }}^{2}\right)>\frac{{\mu }_{1}^{2}+{\mu }_{2}^{2}}{2}{\overline{\theta }}^{2},$$

or \({r}_{\sigma }>{r}_{\mu }.\)

With a limited contract, firm 1’s expected standard is \(E\left({a}_{1}\right)={\mu }_{1}\overline{\theta }.\) With a comprehensive contract, firm 1’s standard \({a}_{1}={\mu }_{1}\overline{\theta }\). Hence, the two standards are equal ex-ante.

1.8 Proof of Proposition 3

It is obvious that \({W}_{l}^{S}>{W}_{l}^{A}\) and \({W}_{c}^{S}={W}_{c}^{A}\).

1.9 Proof of Lemma 4

By \(FO{C}_{1}\) and \(FO{C}_{2}\), we have

$${R}_{h}^{\mathrm{^{\prime}}}\left(\theta ,h\left(a\right)\right)=\frac{{c}_{1}^{\mathrm{^{\prime}}}\left({a}_{1}\right)}{{h}_{1}^{\mathrm{^{\prime}}}\left(a\right)}+\frac{{c}_{2}^{\mathrm{^{\prime}}}\left({a}_{2}\right)}{{h}_{2}^{\mathrm{^{\prime}}}\left(a\right)}.$$

By \(FO{C}_{3}\) and \(FO{C}_{4}\), we have

$$\iint xf\left(x,\theta ,h\left(\widehat{a}\left(\theta \right)\right)\right)dx{f}_{g}^{\mathrm{^{\prime}}}\left(\theta ;g\left(b\right)\right)d\theta =\frac{{C}_{1}^{\mathrm{^{\prime}}}\left({b}_{1}\right)}{{g}_{1}^{\mathrm{^{\prime}}}\left(b\right)}+\frac{{C}_{2}^{\mathrm{^{\prime}}}\left({b}_{2}\right)}{{g}_{2}^{\mathrm{^{\prime}}}\left(b\right)},$$

implying

$$\int R\left(\theta ,h\left(\widehat{a}\left(\theta \right)\right)\right){f}_{g}^{\mathrm{^{\prime}}}\left(\theta ;g\left(b\right)\right)d\theta =\frac{{C}_{1}^{\mathrm{^{\prime}}}\left({b}_{1}\right)}{{g}_{1}^{\mathrm{^{\prime}}}\left(b\right)}+\frac{{C}_{2}^{\mathrm{^{\prime}}}\left({b}_{2}\right)}{{g}_{2}^{\mathrm{^{\prime}}}\left(b\right)},$$

implying

$${R}_{g}^{\mathrm{^{\prime}}}\left(h\left(\widehat{a}\right),g\left({b}_{1},{b}_{2}\right)\right)=\frac{{C}_{1}^{\mathrm{^{\prime}}}\left({b}_{1}\right)}{{g}_{1}^{\mathrm{^{\prime}}}\left(b\right)}+\frac{{C}_{2}^{\mathrm{^{\prime}}}\left({b}_{2}\right)}{{g}_{2}^{\mathrm{^{\prime}}}\left(b\right)}.$$

Then, problem becomes

$$\begin{aligned}&\underset{{s}_{i}\in S,{\widehat{a}}_{i}\in \mathcal{A},{b}_{i}\in {\mathbb{B}}}{{\text{max}}}\int \left(R\left(\theta ,h\left(\widehat{a}\left(\theta \right)\right)\right)-{c}_{1}\left({\widehat{a}}_{1}\left(\theta \right)\right)-{c}_{2}\left({\widehat{a}}_{2}\left(\theta \right)\right)\right)f\left(\theta ;g\left({b}_{1},{b}_{2}\right)\right)d\theta -{C}_{1}\left({b}_{1}\right)-{C}_{2}\left({b}_{2}\right)\\ &{\text{ s.t. }} {R}_{h}^{\mathrm{^{\prime}}}\left(\theta ,h\left(a\right)\right)=\frac{{c}_{1}^{\mathrm{^{\prime}}}\left({a}_{1}\right)}{{h}_{1}^{\mathrm{^{\prime}}}\left(a\right)}+\frac{{c}_{2}^{\mathrm{^{\prime}}}\left({a}_{2}\right)}{{h}_{2}^{\mathrm{^{\prime}}}\left(a\right)},\\ & \quad {R}_{g}^{\mathrm{^{\prime}}}\left(h\left(\widehat{a}\right),g\left({b}_{1},{b}_{2}\right)\right)=\frac{{C}_{1}^{\mathrm{^{\prime}}}\left({b}_{1}\right)}{{g}_{1}^{\mathrm{^{\prime}}}\left(b\right)}+\frac{{C}_{2}^{\mathrm{^{\prime}}}\left({b}_{2}\right)}{{g}_{2}^{\mathrm{^{\prime}}}\left(b\right)},\\ & \quad FO{C}_{2}, FO{C}_{4}, SO{C}_{1}, SO{C}_{2}, SO{C}_{3}, SO{C}_{4},\\ & \quad {s}_{1}\left(x\right)+{s}_{2}\left(x\right)=x,\text{ for all }x\in {\mathbb{R}}_{+}.\end{aligned}$$
(27)

This problem can be solved in two steps. First, solve for \(\left({\widehat{a}}_{1}\left(\theta \right),{\widehat{a}}_{2}\left(\theta \right),{b}_{1},{b}_{2}\right)\) from

$$\begin{array}{l}\underset{{\widehat{a}}_{i}\in \mathcal{A},{b}_{i}\in {\mathbb{B}}}{{\text{max}}}\int \left(R\left(\theta ,h\left(\widehat{a}\left(\theta \right)\right)\right)-{c}_{1}\left({\widehat{a}}_{1}\right)-{c}_{2}\left({\widehat{a}}_{2}\right)\right)f\left(\theta ;g\left({b}_{1},{b}_{2}\right)\right)d\theta -{C}_{1}\left({b}_{1}\right)-{C}_{2}\left({b}_{2}\right)\\ {\text{ s. t.}} {R}_{h}^{\mathrm{^{\prime}}}\left(\theta ,h\left(a\right)\right)=\frac{{c}_{1}^{\mathrm{^{\prime}}}\left({a}_{1}\right)}{{h}_{1}^{\mathrm{^{\prime}}}\left(a\right)}+\frac{{c}_{2}^{\mathrm{^{\prime}}}\left({a}_{2}\right)}{{h}_{2}^{\mathrm{^{\prime}}}\left(a\right)},\\ {R}_{g}^{\mathrm{^{\prime}}}\left(h\left(\widehat{a}\right),g\left({b}_{1},{b}_{2}\right)\right)=\frac{{C}_{1}^{\mathrm{^{\prime}}}\left({b}_{1}\right)}{{g}_{1}^{\mathrm{^{\prime}}}\left(b\right)}+\frac{{C}_{2}^{\mathrm{^{\prime}}}\left({b}_{2}\right)}{{g}_{2}^{\mathrm{^{\prime}}}\left(b\right)}.\end{array}$$

This problem can be further divided into two steps: solve for \(\left({\widehat{a}}_{1}\left(\theta \right),{\widehat{a}}_{2}\left(\theta \right)\right)\) from

$$\begin{array}{l}\underset{{a}_{i}\in {\mathbb{A}}}{{\text{max}}} R\left(\theta ,h\left({a}_{1},{a}_{2}\right)\right)-{c}_{1}\left({a}_{1}\right)-{c}_{2}\left({a}_{2}\right)\\ {\text{s.t. }} {R}_{h}^{\mathrm{^{\prime}}}\left(\theta ,h\left(a\right)\right)=\frac{{c}_{1}^{\mathrm{^{\prime}}}\left({a}_{1}\right)}{{h}_{1}^{\mathrm{^{\prime}}}\left(a\right)}+\frac{{c}_{2}^{\mathrm{^{\prime}}}\left({a}_{2}\right)}{{h}_{2}^{\mathrm{^{\prime}}}\left(a\right)},\end{array}$$

and then, given \(\left({\widehat{a}}_{1}\left(\theta \right),{\widehat{a}}_{2}\left(\theta \right)\right)\), solve for \(\left({b}_{1},{b}_{2}\right)\) from

$$\begin{array}{l}\underset{{b}_{i}\in {\mathbb{B}}}{{\text{max}}} \int \left(R\left(\theta ,h\left(\widehat{a}\left(\theta \right)\right)\right)-{c}_{1}\left({\widehat{a}}_{1}\left(\theta \right)\right)-{c}_{2}\left({\widehat{a}}_{2}\left(\theta \right)\right)\right)f\left(\theta ;g\left({b}_{1},{b}_{2}\right)\right)d\theta -{C}_{1}\left({b}_{1}\right)-{C}_{2}\left({b}_{2}\right)\\ {\text{s.t. }} {R}_{g}^{\mathrm{^{\prime}}}\left(h\left(\widehat{a}\right),g\left({b}_{1},{b}_{2}\right)\right)=\frac{{C}_{1}^{\mathrm{^{\prime}}}\left({b}_{1}\right)}{{g}_{1}^{\mathrm{^{\prime}}}\left(b\right)}+\frac{{C}_{2}^{\mathrm{^{\prime}}}\left({b}_{2}\right)}{{g}_{2}^{\mathrm{^{\prime}}}\left(b\right)}.\end{array}$$

Second, find a sharing scheme satisfying \(FO{C}_{2}\) and \(FO{C}_{4}\). We can then further verify the SOCs. There are typically many sharing schemes that can satisfy \(FO{C}_{2}\) and \(FO{C}_{4}\). We should try to find a simple sharing scheme so that the SOCs are easy to verify.

1.10 Proof of Lemma 5

The objective function of can be written as

$$\begin{aligned}U &\equiv {r}_{1}{\text{ln}}\left(\iint {s}_{1}\left(x\right)f\left(x;\theta ,h\left(a\right)\right)dxf\left(\theta ;g\left(b\right)\right)d\theta -{c}_{1}\left({a}_{1}\right)-{C}_{1}\left({b}_{1}\right)\right)\\ & \quad +{r}_{2}{\text{ln}}\left(\iint {s}_{2}\left(x\right)f\left(x;\theta ,h\left(a\right)\right)dxf\left(\theta ;g\left(b\right)\right)d\theta -{c}_{2}\left({a}_{2}\right)-{C}_{2}\left({b}_{2}\right)\right).\end{aligned}$$

Then, the problem can be written as

$$\begin{array}{l}\underset{{a}_{i}\in {\mathbb{A}}, \, {b}_{i}\in {\mathbb{B}}}{{\text{max}}} \underset{{s}_{i}\in \mathcal{S}}{{\text{max}}} U\\ \text{s.t. } {s}_{1}\left(x\right)+{s}_{2}\left(x\right)=x,\text{ for all }x\in {\mathbb{R}}_{+}.\end{array}$$
(28)

Given \(\left({a}_{1},{a}_{2},{b}_{1},{b}_{2}\right)\), consider problem

$$\begin{array}{l}\underset{{s}_{i}\in \mathcal{S}}{{\text{max}}} U\\ \text{ s.t. } {s}_{1}\left(x\right)+{s}_{2}\left(x\right)=x,\text{ for all }x\in {\mathbb{R}}_{+}.\end{array}$$
(29)

The FOC is

$$\begin{aligned}0=\frac{\partial U}{\partial {s}_{1}} & =\frac{ {r}_{1}\int f\left(x;\theta ,h\left(a\right)\right)f\left(\theta ;g\left(b\right)\right)d\theta }{\iint {s}_{1}\left(x\right)f\left(x;\theta ,h\left(a\right)\right)dxf\left(\theta ;g\left(b\right)\right)d\theta -{c}_{1}\left({a}_{1}\right)-{C}_{1}\left({b}_{1}\right)}\\ &\quad -\frac{{r}_{2}\int f\left(x;\theta ,h\left(a\right)\right)f\left(\theta ;g\left(b\right)\right)d\theta }{\iint {s}_{2}\left(x\right)f\left(x;\theta ,h\left(a\right)\right)dxf\left(\theta ;g\left(b\right)\right)d\theta -{c}_{2}\left({a}_{2}\right)-{C}_{2}\left({b}_{2}\right)},\end{aligned}$$

implying

$$\begin{array}{l}{r}_{1}\left[R\left(h\left(a\right),g\left(b\right)\right)-\iint {s}_{1}\left(x\right)f\left(x;\theta ,h\left(a\right)\right)dxf\left(\theta ;g\left(b\right)\right)d\theta -{c}_{2}\left({a}_{2}\right)-{C}_{2}\left({b}_{2}\right)\right]\\ \quad ={r}_{2}\left[\iint {s}_{1}\left(x\right)f\left(x;\theta ,h\left(a\right)\right)dxf\left(\theta ;g\left(b\right)\right)d\theta -{c}_{1}\left({a}_{1}\right)-{C}_{1}\left({b}_{1}\right)\right],\end{array}$$

implying

$$\iint {s}_{1}\left(x\right)f\left(x;\theta ,h\left(a\right)\right)dxf\left(\theta ;g\left(b\right)\right)d\theta =\frac{{r}_{1}R\left(h\left(a\right),g\left(b\right)\right)+{r}_{2}{c}_{1}\left({a}_{1}\right)+{r}_{2}{C}_{1}\left({b}_{1}\right)-{r}_{1}{c}_{2}\left({a}_{2}\right)-{r}_{1}{C}_{2}\left({b}_{2}\right)}{{r}_{1}+{r}_{2}}.$$
(30)

That is, given\(\left({a}_{1},{a}_{2},{b}_{1},{b}_{2}\right)\), any admissible \({s}_{1}\left(\cdot \right)\) satisfying the above equation is optimal. Consider\({s}_{1}\left(x\right)={\alpha }_{1}x\). Then, by, we have

$${\alpha }_{1}=\frac{{r}_{1}R\left(h\left(a\right),g\left(b\right)\right)+{r}_{2}{c}_{1}\left({a}_{1}\right)+{r}_{2}{C}_{1}\left({b}_{1}\right)-{r}_{1}{c}_{2}\left({a}_{2}\right)-{r}_{1}{C}_{2}\left({b}_{2}\right)}{\left({r}_{1}+{r}_{2}\right)R\left(h\left(a\right),g\left(b\right)\right)}.$$

That is, we find a simple sharing scheme \({s}_{1}\left(x\right)={\alpha }_{1}x\) that satisfies. Then,

$${\alpha }_{2}=1-{\alpha }_{1}=\frac{{r}_{2}R\left(h\left(a\right),g\left(b\right)\right)-{r}_{2}{c}_{1}\left({a}_{1}\right)-{r}_{2}{C}_{1}\left({b}_{1}\right)+{r}_{1}{c}_{2}\left({a}_{2}\right)+{r}_{1}{C}_{2}\left({b}_{2}\right)}{\left({r}_{1}+{r}_{2}\right)R\left(h\left(a\right),g\left(b\right)\right)}.$$

Then,

$$\begin{array}{l}W\left(a,b\right)={\left({\alpha }_{1}R\left(h\left(a\right),g\left(b\right)\right)-{c}_{1}\left({a}_{1}\right)-{C}_{1}\left({b}_{1}\right)\right)}^{{r}_{1}}{\left({\alpha }_{2}R\left(h\left(a\right),g\left(b\right)\right)-{c}_{2}\left({a}_{2}\right)-{C}_{2}\left({b}_{2}\right)\right)}^{{r}_{2}}\\ ={\left(\frac{{r}_{1}R\left(h\left(a\right),g\left(b\right)\right)+{r}_{2}{c}_{1}\left({a}_{1}\right)+{r}_{2}{C}_{1}\left({b}_{1}\right)-{r}_{1}{c}_{2}\left({a}_{2}\right)-{r}_{1}{C}_{2}\left({b}_{2}\right)}{{r}_{1}+{r}_{2}}-{c}_{1}\left({a}_{1}\right)-{C}_{1}\left({b}_{1}\right)\right)}^{{r}_{1}}\\ \cdot {\left(\frac{{r}_{2}R\left(h\left(a\right),g\left(b\right)\right)-{r}_{2}{c}_{1}\left({a}_{1}\right)-{r}_{2}{C}_{1}\left({b}_{1}\right)+{r}_{1}{c}_{2}\left({a}_{2}\right)+{r}_{1}{C}_{2}\left({b}_{2}\right)}{{r}_{1}+{r}_{2}}-{c}_{2}\left({a}_{2}\right)-{C}_{2}\left({b}_{2}\right)\right)}^{{r}_{2}}\\ ={\left({r}_{1}\frac{R\left(h\left(a\right),g\left(b\right)\right)-{c}_{1}\left({a}_{1}\right)-{c}_{2}\left({a}_{2}\right)-{C}_{1}\left({b}_{1}\right)-{C}_{2}\left({b}_{2}\right)}{{r}_{1}+{r}_{2}}\right)}^{{r}_{1}}\\ \cdot {\left({r}_{2}\frac{R\left(h\left(a\right),g\left(b\right)\right)-{c}_{1}\left({a}_{1}\right)-{c}_{2}\left({a}_{2}\right)-{C}_{1}\left({b}_{1}\right)-{C}_{2}\left({b}_{2}\right)}{{r}_{1}+{r}_{2}}\right)}^{{r}_{2}}.\end{array}$$

Then, problem becomes

$$\underset{{a}_{i}\in {\mathbb{A}},{b}_{i}\in {\mathbb{B}}}{{\text{max}}} R\left(h\left(a\right),g\left(b\right)\right)-{c}_{1}\left({a}_{1}\right)-{c}_{2}\left({a}_{2}\right)-{C}_{1}\left({b}_{1}\right)-{C}_{2}\left({b}_{2}\right).$$

1.11 Derivation of Case 3A

We have

$$R\left(\theta ,h\right)=E\left(\widetilde{A}\theta h\right)=\theta h,$$

and

$$R\left(h\left(\widehat{a}\right),g\right)={E}_{\theta }\left(R\left(\theta ,h(\widehat{a}\left(\theta \right)\right)\right)={E}_{\theta }\left(\theta h(\widehat{a}\left(\theta \right)\right).$$

Then, problem becomes

$$\begin{array}{l}\underset{ {a}_{1,}{a}_{2}\in {\mathbb{A}}}{\text{max}} \theta h\left({a}_{1},{a}_{2}\right)-\frac{1}{2}{a}_{1}^{2}-\frac{1}{2}{a}_{2}^{2}\\ \text{ s.t. } \theta =\frac{{a}_{1}}{{\mu }_{1}}+\frac{{a}_{2}}{{\mu }_{2}}.\end{array}$$

The solution is

$${\widehat{a}}_{1}\left(\theta \right)=\frac{{\mu }_{1}^{3}}{{\mu }_{1}^{2}+{\mu }_{2}^{2}}\theta , {\widehat{a}}_{2}\left(\theta \right)=\frac{{\mu }_{2}^{3}}{{\mu }_{1}^{2}+{\mu }_{2}^{2}}\theta .$$

Then,

$$\begin{array}{l}{U}_{l}^{Q}={E}_{\theta }\left(R\left(\theta ,h\left(\widehat{a}\left(\theta \right)\right)\right)-{c}_{1}\left({\widehat{a}}_{1}\left(\theta \right)\right)-{c}_{2}\left({\widehat{a}}_{2}\left(\theta \right)\right)\right)\\ ={E}_{\theta }\left(\theta \left(\frac{{\mu }_{1}^{4}}{{\mu }_{1}^{2}+{\mu }_{2}^{2}}\theta +\frac{{\mu }_{2}^{4}}{{\mu }_{1}^{2}+{\mu }_{2}^{2}}\theta \right)-\frac{1}{2}{\left(\frac{{\mu }_{1}^{3}}{{\mu }_{1}^{2}+{\mu }_{2}^{2}}\theta \right)}^{2}-\frac{1}{2}{\left(\frac{{\mu }_{2}^{3}}{{\mu }_{1}^{2}+{\mu }_{2}^{2}}\theta \right)}^{2}\right)\\ =\frac{{E}_{\theta }\left({\theta }^{2}\right)}{2{\left({\mu }_{1}^{2}+{\mu }_{2}^{2}\right)}^{2}}\left[2\left({\mu }_{1}^{2}+{\mu }_{2}^{2}\right)\left({\mu }_{1}^{4}+{\mu }_{2}^{4}\right)-{\mu }_{1}^{6}-{\mu }_{2}^{6}\right]\\ =\frac{{\mu }_{1}^{4}+{\mu }_{2}^{4}+{\mu }_{1}^{2}{\mu }_{2}^{2}}{2\left({\mu }_{1}^{2}+{\mu }_{2}^{2}\right)}{E}_{\theta }\left({\theta }^{2}\right).\end{array}$$

We have

$$E\left({\widetilde{\theta }}^{2}\right)=g\left(b\right)E\left({\widetilde{B}}^{2}\right)=g\left(b\right)\left({\sigma }_{B}^{2}+{\overline{B} }^{2}\right),$$

and

$$\begin{aligned}R\left(h\left(\widehat{a}\right),g\right)&={E}_{\theta }\left[\theta h\left(\widehat{a}\left(\theta \right)\right)\right]={E}_{\theta }\left[\theta \left(\frac{{\mu }_{1}^{4}}{{\mu }_{1}^{2}+{\mu }_{2}^{2}}\theta +\frac{{\mu }_{2}^{4}}{{\mu }_{1}^{2}+{\mu }_{2}^{2}}\theta \right)\right]\\ \quad & =\frac{{\mu }_{1}^{4}+{\mu }_{2}^{4}}{{\mu }_{1}^{2}+{\mu }_{2}^{2}}{E}_{\theta }\left({\theta }^{2}\right)=\frac{{\mu }_{1}^{4}+{\mu }_{2}^{4}}{{\mu }_{1}^{2}+{\mu }_{2}^{2}}\left({\sigma }_{B}^{2}+{\overline{B} }^{2}\right)g.\end{aligned}$$

Then, problem becomes

$$\begin{array}{l}{W}_{l}^{Q}=\underset{{b}_{i}\in {\mathbb{B}}}{{\text{max}}} \frac{{\mu }_{1}^{4}+{\mu }_{2}^{4}+{\mu }_{1}^{2}{\mu }_{2}^{2}}{2\left({\mu }_{1}^{2}+{\mu }_{2}^{2}\right)}\left({\sigma }_{B}^{2}+{\overline{B} }^{2}\right)\left({\beta }_{1}{b}_{1}+{\beta }_{2}{b}_{2}\right)-\frac{1}{2}{b}_{1}^{2}-\frac{1}{2}{b}_{2}^{2}\\ \text{ s.t. } \frac{{\mu }_{1}^{4}+{\mu }_{2}^{4}}{{\mu }_{1}^{2}+{\mu }_{2}^{2}}\left({\sigma }_{B}^{2}+{\overline{B} }^{2}\right)=\frac{{b}_{1}}{{\beta }_{1}}+\frac{{b}_{2}}{{\beta }_{2}}.\end{array}$$

The solution is

$$\begin{array}{c}{b}_{1}={\beta }_{1}\frac{\left({\mu }_{1}^{4}+{\mu }_{2}^{4}\right)\left({\beta }_{1}^{2}+{\beta }_{2}^{2}\right)+{\mu }_{1}^{2}{\mu }_{2}^{2}\left({\beta }_{1}^{2}-{\beta }_{2}^{2}\right)}{2\left({\mu }_{1}^{2}+{\mu }_{2}^{2}\right)\left({\beta }_{1}^{2}+{\beta }_{2}^{2}\right)}\left({\sigma }_{B}^{2}+{\overline{B} }^{2}\right),\\ {b}_{2}={\beta }_{2}\frac{\left({\mu }_{1}^{4}+{\mu }_{2}^{4}\right)\left({\beta }_{1}^{2}+{\beta }_{2}^{2}\right)+{\mu }_{1}^{2}{\mu }_{2}^{2}\left({\beta }_{2}^{2}-{\beta }_{1}^{2}\right)}{2\left({\mu }_{1}^{2}+{\mu }_{2}^{2}\right)\left({\beta }_{1}^{2}+{\beta }_{2}^{2}\right)}\left({\sigma }_{B}^{2}+{\overline{B} }^{2}\right).\end{array}$$

Then,

$$\begin{array}{c}{\beta }_{1}{b}_{1}+{\beta }_{2}{b}_{2}=\frac{\left({\mu }_{1}^{4}+{\mu }_{2}^{4}\right){\left({\beta }_{1}^{2}+{\beta }_{2}^{2}\right)}^{2}+{\mu }_{1}^{2}{\mu }_{2}^{2}{\left({\beta }_{1}^{2}-{\beta }_{2}^{2}\right)}^{2}}{2\left({\mu }_{1}^{2}+{\mu }_{2}^{2}\right)\left({\beta }_{1}^{2}+{\beta }_{2}^{2}\right)}\left({\sigma }_{B}^{2}+{\overline{B} }^{2}\right).\end{array}$$

Then,

$$\overline{\theta }=\overline{B}\sqrt{g }=\overline{B }\sqrt{\frac{\left({\mu }_{1}^{4}+{\mu }_{2}^{4}\right){\left({\beta }_{1}^{2}+{\beta }_{2}^{2}\right)}^{2}+{\mu }_{1}^{2}{\mu }_{2}^{2}{\left({\beta }_{1}^{2}-{\beta }_{2}^{2}\right)}^{2}}{2\left({\mu }_{1}^{2}+{\mu }_{2}^{2}\right)\left({\beta }_{1}^{2}+{\beta }_{2}^{2}\right)}\left({\sigma }_{B}^{2}+{\overline{B} }^{2}\right)} ,$$

and

$$\begin{aligned}{W}_{l}^{Q}& =\frac{{\mu }_{1}^{4}+{\mu }_{1}^{2}{\mu }_{2}^{2}+{\mu }_{2}^{4}}{2\left({\mu }_{1}^{2}+{\mu }_{2}^{2}\right)}\frac{\left({\mu }_{1}^{4}+{\mu }_{2}^{4}\right){\left({\beta }_{1}^{2}+{\beta }_{2}^{2}\right)}^{2}+{\mu }_{1}^{2}{\mu }_{2}^{2}{\left({\beta }_{1}^{2}-{\beta }_{2}^{2}\right)}^{2}}{2\left({\mu }_{1}^{2}+{\mu }_{2}^{2}\right)\left({\beta }_{1}^{2}+{\beta }_{2}^{2}\right)}{\left({\sigma }_{B}^{2}+{\overline{B} }^{2}\right)}^{2}\\ &\quad -\frac{1}{2}{\left({\beta }_{1}\frac{\left({\mu }_{1}^{4}+{\mu }_{2}^{4}\right)\left({\beta }_{1}^{2}+{\beta }_{2}^{2}\right)+{\mu }_{1}^{2}{\mu }_{2}^{2}\left({\beta }_{1}^{2}-{\beta }_{2}^{2}\right)}{2\left({\mu }_{1}^{2}+{\mu }_{2}^{2}\right)\left({\beta }_{1}^{2}+{\beta }_{2}^{2}\right)}\right)}^{2}{\left({\sigma }_{B}^{2}+{\overline{B} }^{2}\right)}^{2}\\ & \quad -\frac{1}{2}{\left({\beta }_{2}\frac{\left({\mu }_{1}^{4}+{\mu }_{2}^{4}\right)\left({\beta }_{1}^{2}+{\beta }_{2}^{2}\right)+{\mu }_{1}^{2}{\mu }_{2}^{2}\left({\beta }_{2}^{2}-{\beta }_{1}^{2}\right)}{2\left({\mu }_{1}^{2}+{\mu }_{2}^{2}\right)\left({\beta }_{1}^{2}+{\beta }_{2}^{2}\right)}\right)}^{2}{\left({\sigma }_{B}^{2}+{\overline{B} }^{2}\right)}^{2}.\end{aligned}$$

Since

$$\begin{aligned}& 2\left({\beta }_{1}^{2}+{\beta }_{2}^{2}\right)\left({\mu }_{1}^{4}+{\mu }_{1}^{2}{\mu }_{2}^{2}+{\mu }_{2}^{4}\right)\left(\left({\mu }_{1}^{4}+{\mu }_{2}^{4}\right){\left({\beta }_{1}^{2}+{\beta }_{2}^{2}\right)}^{2}+{\mu }_{1}^{2}{\mu }_{2}^{2}{\left({\beta }_{1}^{2}-{\beta }_{2}^{2}\right)}^{2}\right)\\ & \quad -{\beta }_{1}^{2}{\left[\left({\mu }_{1}^{4}+{\mu }_{2}^{4}\right)\left({\beta }_{1}^{2}+{\beta }_{2}^{2}\right)+{\mu }_{1}^{2}{\mu }_{2}^{2}\left({\beta }_{1}^{2}-{\beta }_{2}^{2}\right)\right]}^{2}\\ & \quad -{\beta }_{2}^{2}{\left[\left({\mu }_{1}^{4}+{\mu }_{2}^{4}\right)\left({\beta }_{1}^{2}+{\beta }_{2}^{2}\right)+{\mu }_{1}^{2}{\mu }_{2}^{2}\left({\beta }_{2}^{2}-{\beta }_{1}^{2}\right)\right]}^{2}\\ & \quad =\left({\beta }_{1}^{2}+{\beta }_{2}^{2}\right)\left({\left({\beta }_{1}^{2}+{\beta }_{2}^{2}\right)}^{2}{\left({\mu }_{1}^{4}+{\mu }_{1}^{2}{\mu }_{2}^{2}+{\mu }_{2}^{4}\right)}^{2}-4{\beta }_{1}^{2}{\beta }_{2}^{2}{\mu }_{1}^{4}{\mu }_{2}^{4}\right),\end{aligned}$$

we have

$${W}_{l}^{Q}=\frac{{\left({\beta }_{1}^{2}+{\beta }_{2}^{2}\right)}^{2}{\left({\mu }_{1}^{4}+{\mu }_{1}^{2}{\mu }_{2}^{2}+{\mu }_{2}^{4}\right)}^{2}-4{\beta }_{1}^{2}{\beta }_{2}^{2}{\mu }_{1}^{4}{\mu }_{2}^{4}}{8\left({\beta }_{1}^{2}+{\beta }_{2}^{2}\right){\left({\mu }_{1}^{2}+{\mu }_{2}^{2}\right)}^{2}}{\left({\sigma }_{B}^{2}+{\overline{B} }^{2}\right)}^{2}.$$

For a comprehensive contract, consider problem:

$${W}_{c}^{Q}=\underset{{a}_{i}\in {\mathbb{A}},{b}_{i}\in {\mathbb{B}}}{{\text{max}}} R\left(h\left(a\right),g\left(b\right)\right)-\frac{1}{2}{a}_{1}^{2}-\frac{1}{2}{a}_{2}^{2}-\frac{1}{2}{b}_{1}^{2}-\frac{1}{2}{b}_{2}^{2}.$$

We have

$$R\left(\theta ,h\right)=E\left(\widetilde{A}\theta h\right)=\theta h,$$

and

$$R\left(g,h\right)={E}_{\theta }\left(\theta h\right)=E\left(\widetilde{B}\sqrt{g}h\right)=\overline{B}\sqrt{g }h.$$

Then, the problem becomes

$${W}_{c}^{Q}=\underset{{a}_{i}\in {\mathbb{A}},{b}_{i}\in {\mathbb{B}}}{{\text{max}}} \overline{B}\sqrt{{\beta }_{1}{b}_{1}+{\beta }_{2}{b}_{2}}\left({\mu }_{1}{a}_{1}+{\mu }_{2}{a}_{2}\right)-\frac{1}{2}{a}_{1}^{2}-\frac{1}{2}{a}_{2}^{2}-\frac{1}{2}{b}_{1}^{2}-\frac{1}{2}{b}_{2}^{2}.$$

The FOCs are

$$\begin{array}{l}{\mu }_{1}\overline{B}\sqrt{{\beta }_{1}{b}_{1}+{\beta }_{2}{b}_{2}}={a}_{1},\\ {\mu }_{2}\overline{B}\sqrt{{\beta }_{1}{b}_{1}+{\beta }_{2}{b}_{2}}={a}_{2},\\ {\beta }_{1}\overline{B}\frac{{\mu }_{1}{a}_{1}+{\mu }_{2}{a}_{2}}{2\sqrt{{\beta }_{1}{b}_{1}+{\beta }_{2}{b}_{2}}}={b}_{1},\\ {\beta }_{2}\overline{B}\frac{{\mu }_{1}{a}_{1}+{\mu }_{2}{a}_{2}}{2\sqrt{{\beta }_{1}{b}_{1}+{\beta }_{2}{b}_{2}}}={b}_{2},\end{array}$$

implying

$${b}_{i}={\beta }_{i}{\overline{B} }^{2}\frac{\left({\mu }_{1}^{2}+{\mu }_{2}^{2}\right)\sqrt{{\beta }_{1}{b}_{1}+{\beta }_{2}{b}_{2}}}{2\sqrt{{\beta }_{1}{b}_{1}+{\beta }_{2}{b}_{2}}}=\frac{{\beta }_{i}}{2}{\overline{B} }^{2}\left({\mu }_{1}^{2}+{\mu }_{2}^{2}\right).$$

Then,

$${a}_{i}={\mu }_{i}\overline{B}\sqrt{\frac{{\beta }_{1}^{2}}{2}{\overline{B} }^{2}\left({\mu }_{1}^{2}+{\mu }_{2}^{2}\right)+\frac{{\beta }_{2}^{2}}{2}{\overline{B} }^{2}\left({\mu }_{1}^{2}+{\mu }_{2}^{2}\right)}={\mu }_{i}{\overline{B} }^{2}\sqrt{\frac{({\beta }_{1}^{2}+{\beta }_{2}^{2})\left({\mu }_{1}^{2}+{\mu }_{2}^{2}\right)}{2}}.$$

Then,

$$\overline{\theta }=\overline{B}\sqrt{g }={\overline{B} }^{2}\sqrt{\frac{({\beta }_{1}^{2}+{\beta }_{2}^{2})\left({\mu }_{1}^{2}+{\mu }_{2}^{2}\right)}{2}} ,$$

and

$$\begin{array}{l}R\left(g,h\right)={\overline{B} }^{2}\sqrt{\frac{\left({\beta }_{1}^{2}+{\beta }_{2}^{2}\right)\left({\mu }_{1}^{2}+{\mu }_{2}^{2}\right)}{2}}\left({\mu }_{1}^{2}+{\mu }_{2}^{2}\right){\overline{B} }^{2}\sqrt{\frac{({\beta }_{1}^{2}+{\beta }_{2}^{2})\left({\mu }_{1}^{2}+{\mu }_{2}^{2}\right)}{2}}\\ \end{array}=\frac{1}{2}{\overline{B} }^{4}\left({\beta }_{1}^{2}+{\beta }_{2}^{2}\right){\left({\mu }_{1}^{2}+{\mu }_{2}^{2}\right)}^{2},$$

and

$$\begin{array}{c}{W}_{c}^{Q}=\frac{{\overline{B} }^{4}}{8}\left({\beta }_{1}^{2}+{\beta }_{2}^{2}\right){\left({\mu }_{1}^{2}+{\mu }_{2}^{2}\right)}^{2}.\end{array}$$

1.12 Proof of Proposition 4

For part (a), we isolate the effect of the disparity between \({\beta }_{1}\) and \({\beta }_{2}\) by assuming \({\beta }_{1}={\beta }_{2}\equiv \beta\). Then,

$${W}_{c}^{Q}=\frac{1}{4}{\beta }^{2}{\overline{B} }^{4}{\left({\mu }_{1}^{2}+{\mu }_{2}^{2}\right)}^{2},$$

and

$$\begin{array}{c}{W}_{l}^{Q}=\frac{1}{4}\left({\mu }_{1}^{4}+{\mu }_{2}^{4}\right){\beta }^{2}{\left({\sigma }_{B}^{2}+{\overline{B} }^{2}\right)}^{2}.\end{array}$$

Then, \({W}_{l}^{Q}>{W}_{c}^{Q}\) iff

$${r}_{B}^{2}>\frac{1+{r}_{\mu }^{2}}{\sqrt{1+{r}_{\mu }^{4}}}-1.$$
(31)

Consider

$$\begin{array}{c}\frac{\partial }{\partial z}\left(\frac{{\left(1+z\right)}^{2}}{1+{z}^{2}}\right)=\frac{2\left(1+z\right)(1-z)}{{\left(1+{z}^{2}\right)}^{2}}.\end{array}$$

This implies that the right-hand side of is increasing when \({r}_{\mu }<1\) and decreasing when \({r}_{\mu }>1\), reaching its maximum at \({r}_{\mu }=1\).

For part (b), we isolate the effect of the disparity between \({\mu }_{1}\) and \({\mu }_{2}\) by assuming \({\mu }_{1}={\mu }_{2}\equiv \mu\). Then,

$${W}_{c}^{Q}={\mu }^{4}{\overline{B} }^{4}\left({\beta }_{1}^{2}+{\beta }_{2}^{2}\right)/2,$$

and

$${W}_{l}^{Q}=\frac{9\left({\beta }_{1}^{4}+{\beta }_{2}^{4}\right)+14{\beta }_{1}^{2}{\beta }_{2}^{2}}{32\left({\beta }_{1}^{2}+{\beta }_{2}^{2}\right)}{\mu }^{4}{\left({\sigma }_{B}^{2}+{\overline{B} }^{2}\right)}^{2}.$$

Then, \({W}_{l}^{Q}>{W}_{c}^{Q}\) iff

$${r}_{B}^{2}>\frac{4\left(1+{r}_{\beta }^{2}\right)}{\sqrt{9\left(1+{r}_{\beta }^{4}\right)+14{r}_{\beta }^{2}}}-1.$$
(32)

Consider

$$\frac{\partial }{\partial z}\left(\frac{{\left(1+z\right)}^{2}}{9\left(1+{z}^{2}\right)+14z}\right)=\frac{4(1-{z}^{2})}{{\left[9\left(1+{z}^{2}\right)+14z\right]}^{2}}.$$

This implies that the right-hand side of is increasing when \({r}_{\beta }<1\) and decreasing when \({r}_{\beta }>1\), reaching maximum at \({r}_{\beta }=1\).

1.13 Derivation of Case 3B

We have

$$R\left(\theta ,h\right)=E\left(\widetilde{A}\theta h\right)=\theta h,$$

and

$$R\left(h\left(\widehat{a}\right),g\right)={E}_{\theta }\left(R\left(\theta ,h(\widehat{a}\left(\theta \right)\right)\right)={E}_{\theta }\left(\theta h(\widehat{a}\left(\theta \right)\right).$$

Then, problem becomes

$$\begin{array}{l}\underset{ {a}_{1,}{a}_{2}\in {\mathbb{A}}}{\text{max}} \theta h\left({a}_{1},{a}_{2}\right)-\frac{1}{2}{a}_{1}^{2}-\frac{1}{2}{a}_{2}^{2}\\ \text{ s.t. } \theta =\frac{{a}_{1}}{{\mu }_{1}}+\frac{{a}_{2}}{{\mu }_{2}}.\end{array}$$

The solution is

$${\widehat{a}}_{1}\left(\theta \right)=\frac{{\mu }_{1}^{3}}{{\mu }_{1}^{2}+{\mu }_{2}^{2}}\theta , \,{\widehat{a}}_{2}\left(\theta \right)=\frac{{\mu }_{2}^{3}}{{\mu }_{1}^{2}+{\mu }_{2}^{2}}\theta .$$

Then,

$$\begin{array}{c}{U}_{l}^{Q}={E}_{\theta }\left(R\left(\theta ,h\left(\widehat{a}\left(\theta \right)\right)\right)-{c}_{1}\left({\widehat{a}}_{1}\left(\theta \right)\right)-{c}_{2}\left({\widehat{a}}_{2}\left(\theta \right)\right)\right)=\frac{{\mu }_{1}^{4}+{\mu }_{2}^{4}+{\mu }_{1}^{2}{\mu }_{2}^{2}}{2\left({\mu }_{1}^{2}+{\mu }_{2}^{2}\right)}{E}_{\theta }\left({\theta }^{2}\right).\end{array}$$

We have

$$E\left({\widetilde{\theta }}^{2}\right)=g\left(b\right)E\left({\widetilde{\theta }}_{0}^{2}\right)=g\left(b\right)\left({\sigma }_{{\theta }_{0}}^{2}+{\overline{\theta }}_{0}^{2}\right),$$

and

$$\begin{array}{c}R\left(h\left(\widehat{a}\right),g\right)={E}_{\theta }\left[\theta h\left(\widehat{a}\left(\theta \right)\right)\right]=\frac{{\mu }_{1}^{4}+{\mu }_{2}^{4}}{{\mu }_{1}^{2}+{\mu }_{2}^{2}}{E}_{\theta }\left({\theta }^{2}\right)=\frac{{\mu }_{1}^{4}+{\mu }_{2}^{4}}{{\mu }_{1}^{2}+{\mu }_{2}^{2}}\left({\sigma }_{{\theta }_{0}}^{2}+{\overline{\theta }}_{0}^{2}\right)g.\end{array}$$

Then, problem becomes

$$\begin{array}{l}{W}_{l}^{Q}=\underset{{b}_{i}\in {\mathbb{B}}}{{\text{max}}} \frac{{\mu }_{1}^{4}+{\mu }_{2}^{4}+{\mu }_{1}^{2}{\mu }_{2}^{2}}{2\left({\mu }_{1}^{2}+{\mu }_{2}^{2}\right)}\left({\sigma }_{{\theta }_{0}}^{2}+{\overline{\theta }}_{0}^{2}\right)\left(1+{\beta }_{1}{b}_{1}+{\beta }_{2}{b}_{2}\right)-\frac{1}{2}{b}_{1}^{2}-\frac{1}{2}{b}_{2}^{2}\\ \text{ s.t. } \frac{{\mu }_{1}^{4}+{\mu }_{2}^{4}}{{\mu }_{1}^{2}+{\mu }_{2}^{2}}\left({\sigma }_{{\theta }_{0}}^{2}+{\overline{\theta }}_{0}^{2}\right)=\frac{{b}_{1}}{{\beta }_{1}}+\frac{{b}_{2}}{{\beta }_{2}}.\end{array}$$

The solution is

$$\begin{array}{l}{b}_{1}={\beta }_{1}\frac{\left({\mu }_{1}^{4}+{\mu }_{2}^{4}\right)\left({\beta }_{1}^{2}+{\beta }_{2}^{2}\right)+{\mu }_{1}^{2}{\mu }_{2}^{2}\left({\beta }_{1}^{2}-{\beta }_{2}^{2}\right)}{2\left({\mu }_{1}^{2}+{\mu }_{2}^{2}\right)\left({\beta }_{1}^{2}+{\beta }_{2}^{2}\right)}\left({\sigma }_{{\theta }_{0}}^{2}+{\overline{\theta }}_{0}^{2}\right),\\ {b}_{2}={\beta }_{2}\frac{\left({\mu }_{1}^{4}+{\mu }_{2}^{4}\right)\left({\beta }_{1}^{2}+{\beta }_{2}^{2}\right)+{\mu }_{1}^{2}{\mu }_{2}^{2}\left({\beta }_{2}^{2}-{\beta }_{1}^{2}\right)}{2\left({\mu }_{1}^{2}+{\mu }_{2}^{2}\right)\left({\beta }_{1}^{2}+{\beta }_{2}^{2}\right)}\left({\sigma }_{{\theta }_{0}}^{2}+{\overline{\theta }}_{0}^{2}\right).\end{array}$$

Then,

$$\begin{array}{c}{\beta }_{1}{b}_{1}+{\beta }_{2}{b}_{2}=\frac{\left({\mu }_{1}^{4}+{\mu }_{2}^{4}\right){\left({\beta }_{1}^{2}+{\beta }_{2}^{2}\right)}^{2}+{\mu }_{1}^{2}{\mu }_{2}^{2}{\left({\beta }_{1}^{2}-{\beta }_{2}^{2}\right)}^{2}}{2\left({\mu }_{1}^{2}+{\mu }_{2}^{2}\right)\left({\beta }_{1}^{2}+{\beta }_{2}^{2}\right)}\left({\sigma }_{{\theta }_{0}}^{2}+{\overline{\theta }}_{0}^{2}\right).\end{array}$$

Then,

$$\overline{\theta }={\overline{\theta }}_{0}\sqrt{g}={\overline{\theta }}_{0}\sqrt{1+\frac{\left({\mu }_{1}^{4}+{\mu }_{2}^{4}\right){\left({\beta }_{1}^{2}+{\beta }_{2}^{2}\right)}^{2}+{\mu }_{1}^{2}{\mu }_{2}^{2}{\left({\beta }_{1}^{2}-{\beta }_{2}^{2}\right)}^{2}}{2\left({\mu }_{1}^{2}+{\mu }_{2}^{2}\right)\left({\beta }_{1}^{2}+{\beta }_{2}^{2}\right)}\left({\sigma }_{{\theta }_{0}}^{2}+{\overline{\theta }}_{0}^{2}\right)} ,$$

and

$${W}_{l}^{Q}=\frac{{\mu }_{1}^{4}+{\mu }_{1}^{2}{\mu }_{2}^{2}+{\mu }_{2}^{4}}{2\left({\mu }_{1}^{2}+{\mu }_{2}^{2}\right)}\left({\sigma }_{{\theta }_{0}}^{2}+{\overline{\theta }}_{0}^{2}\right)+\frac{{\left({\beta }_{1}^{2}+{\beta }_{2}^{2}\right)}^{2}{\left({\mu }_{1}^{4}+{\mu }_{1}^{2}{\mu }_{2}^{2}+{\mu }_{2}^{4}\right)}^{2}-4{\beta }_{1}^{2}{\beta }_{2}^{2}{\mu }_{1}^{4}{\mu }_{2}^{4}}{8\left({\beta }_{1}^{2}+{\beta }_{2}^{2}\right){\left({\mu }_{1}^{2}+{\mu }_{2}^{2}\right)}^{2}}{\left({\sigma }_{{\theta }_{0}}^{2}+{\overline{\theta }}_{0}^{2}\right)}^{2}.$$

For a comprehensive contract, consider problem:

$${W}_{c}^{Q}=\underset{{a}_{i}\in {\mathbb{A}},{b}_{i}\in {\mathbb{B}}}{{\text{max}}} R\left(h\left(a\right),g\left(b\right)\right)-\frac{1}{2}{a}_{1}^{2}-\frac{1}{2}{a}_{2}^{2}-\frac{1}{2}{b}_{1}^{2}-\frac{1}{2}{b}_{2}^{2}.$$

We have

$$R\left(\theta ,h\right)=E\left(\widetilde{A}\theta h\right)=\theta h,$$

and

$$R\left(g,h\right)={E}_{\theta }\left(\theta h\right)=E\left({\widetilde{\theta }}_{0}\sqrt{g}h\right)={\overline{\theta }}_{0}\sqrt{g}h.$$

Then, the problem becomes

$${W}_{c}^{Q}=\underset{{a}_{i}\in {\mathbb{A}},{b}_{i}\in {\mathbb{B}}}{{\text{max}}} {\overline{\theta }}_{0}\sqrt{1+{\beta }_{1}{b}_{1}+{\beta }_{2}{b}_{2}}\left({\mu }_{1}{a}_{1}+{\mu }_{2}{a}_{2}\right)-\frac{1}{2}{a}_{1}^{2}-\frac{1}{2}{a}_{2}^{2}-\frac{1}{2}{b}_{1}^{2}-\frac{1}{2}{b}_{2}^{2}.$$

The FOCs are

$$\begin{array}{l}{\mu }_{1}{\overline{\theta }}_{0}\sqrt{1+{\beta }_{1}{b}_{1}+{\beta }_{2}{b}_{2}}={a}_{1},\\ {\mu }_{2}{\overline{\theta }}_{0}\sqrt{1+{\beta }_{1}{b}_{1}+{\beta }_{2}{b}_{2}}={a}_{2},\\ {\beta }_{1}{\overline{\theta }}_{0}\frac{{\mu }_{1}{a}_{1}+{\mu }_{2}{a}_{2}}{2\sqrt{1+{\beta }_{1}{b}_{1}+{\beta }_{2}{b}_{2}}}={b}_{1},\\ {\beta }_{2}{\overline{\theta }}_{0}\frac{{\mu }_{1}{a}_{1}+{\mu }_{2}{a}_{2}}{2\sqrt{1+{\beta }_{1}{b}_{1}+{\beta }_{2}{b}_{2}}}={b}_{2},\end{array}$$

implying

$${b}_{i}=\frac{{\beta }_{i}}{2}{\overline{\theta }}_{0}^{2}\left({\mu }_{1}^{2}+{\mu }_{2}^{2}\right).$$

Then,

$${a}_{{\text{i}}}={\mu }_{i}{\overline{\theta }}_{0}\sqrt{1+\frac{1}{2}({\beta }_{1}^{2}+{\beta }_{2}^{2})\left({\mu }_{1}^{2}+{\mu }_{2}^{2}\right){\overline{\theta }}_{0}^{2}}.$$

Then,

$$\overline{\theta }={\overline{\theta }}_{0}\sqrt{g}={\overline{\theta }}_{0}\sqrt{1+\frac{1}{2}({\beta }_{1}^{2}+{\beta }_{2}^{2})\left({\mu }_{1}^{2}+{\mu }_{2}^{2}\right){\overline{\theta }}_{0}^{2}} ,$$

and

$$\begin{array}{c}R\left(g,h\right)=\left({\mu }_{1}^{2}+{\mu }_{2}^{2}\right){\overline{\theta }}_{0}^{2}+\frac{1}{2}\left({\beta }_{1}^{2}+{\beta }_{2}^{2}\right){\left({\mu }_{1}^{2}+{\mu }_{2}^{2}\right)}^{2}{\overline{\theta }}_{0}^{4},\end{array}$$

and

$$\begin{array}{c}{W}_{c}^{Q}=\frac{1}{2}\left({\mu }_{1}^{2}+{\mu }_{2}^{2}\right){\overline{\theta }}_{0}^{2}+\frac{1}{8}\left({\beta }_{1}^{2}+{\beta }_{2}^{2}\right){\left({\mu }_{1}^{2}+{\mu }_{2}^{2}\right)}^{2}{\overline{\theta }}_{0}^{4}.\end{array}$$

1.14 Proof of Proposition 5

For part (a), since

$${\beta }_{1}^{2}+{\beta }_{2}^{2}>2{\beta }_{1}{\beta }_{2}$$

and

$${\mu }_{1}^{4}+{\mu }_{1}^{2}{\mu }_{2}^{2}+{\mu }_{2}^{4}>{\mu }_{1}^{4}+{\mu }_{2}^{4}>2{\mu }_{1}^{2}{\mu }_{2}^{2},$$

we have

$${\left({\beta }_{1}^{2}+{\beta }_{2}^{2}\right)}^{2}{\left({\mu }_{1}^{4}+{\mu }_{1}^{2}{\mu }_{2}^{2}+{\mu }_{2}^{4}\right)}^{2}>4{\beta }_{1}^{2}{\beta }_{2}^{2}4{\mu }_{1}^{4}{\mu }_{2}^{4},$$

implying \({W}_{l}^{Q}>{W}_{l}^{S}\). We also obviously have \({W}_{c}^{Q}>{W}_{c}^{S}\).

For part (b), consider inequality \({W}_{l}^{Q}>{W}_{c}^{Q}\), or

$$\begin{aligned}&\frac{4}{{\beta }_{1}^{2}+{\beta }_{2}^{2}}\left({\sigma }_{{\theta }_{0}}^{2}+{\overline{\theta }}_{0}^{2}\right)\frac{{\mu }_{1}^{6}-{\mu }_{2}^{6}}{{\mu }_{1}^{4}-{\mu }_{2}^{4}}+\frac{{\left({\beta }_{1}^{2}+{\beta }_{2}^{2}\right)}^{2}{\left({\mu }_{1}^{4}+{\mu }_{1}^{2}{\mu }_{2}^{2}+{\mu }_{2}^{4}\right)}^{2}-4{\beta }_{1}^{2}{\beta }_{2}^{2}{\mu }_{1}^{4}{\mu }_{2}^{4}}{{\left({\beta }_{1}^{2}+{\beta }_{2}^{2}\right)}^{2}{\left({\mu }_{1}^{2}+{\mu }_{2}^{2}\right)}^{2}}{\left({\sigma }_{{\theta }_{0}}^{2}+{\overline{\theta }}_{0}^{2}\right)}^{2}\\ & \quad >\frac{4}{{\beta }_{1}^{2}+{\beta }_{2}^{2}}\left({\mu }_{1}^{2}+{\mu }_{2}^{2}\right){\overline{\theta }}_{0}^{2}+{\left({\mu }_{1}^{2}+{\mu }_{2}^{2}\right)}^{2}{\overline{\theta }}_{0}^{4},\end{aligned}$$

or

$$\begin{aligned}&\frac{4}{{\beta }_{1}^{2}+{\beta }_{2}^{2}}\left({\sigma }_{{\theta }_{0}}^{2}+{\overline{\theta }}_{0}^{2}\right)\frac{{\mu }_{1}^{6}-{\mu }_{2}^{6}}{{\mu }_{1}^{4}-{\mu }_{2}^{4}}+{\left(\left({\sigma }_{{\theta }_{0}}^{2}+{\overline{\theta }}_{0}^{2}\right)\frac{{\mu }_{1}^{6}-{\mu }_{2}^{6}}{{\mu }_{1}^{4}-{\mu }_{2}^{4}}\right)}^{2}-\frac{4{\beta }_{1}^{2}{\beta }_{2}^{2}{\mu }_{1}^{4}{\mu }_{2}^{4}}{{\left({\beta }_{1}^{2}+{\beta }_{2}^{2}\right)}^{2}{\left({\mu }_{1}^{2}+{\mu }_{2}^{2}\right)}^{2}}{\left({\sigma }_{{\theta }_{0}}^{2}+{\overline{\theta }}_{0}^{2}\right)}^{2}\\ & \quad >{\left(\left({\mu }_{1}^{2}+{\mu }_{2}^{2}\right){\overline{\theta }}_{0}^{2}+\frac{2}{{\beta }_{1}^{2}+{\beta }_{2}^{2}}\right)}^{2}-{\left(\frac{2}{{\beta }_{1}^{2}+{\beta }_{2}^{2}}\right)}^{2},\end{aligned}$$

or

$$\begin{aligned}&{\left(\left({\sigma }_{{\theta }_{0}}^{2}+{\overline{\theta }}_{0}^{2}\right)\frac{{\mu }_{1}^{6}-{\mu }_{2}^{6}}{{\mu }_{1}^{4}-{\mu }_{2}^{4}}+\frac{2}{{\beta }_{1}^{2}+{\beta }_{2}^{2}}\right)}^{2}-{\left(\frac{2}{{\beta }_{1}^{2}+{\beta }_{2}^{2}}\right)}^{2}-\frac{4{\gamma }_{1}^{2}{\gamma }_{2}^{2}{\mu }_{1}^{4}{\mu }_{2}^{4}}{{\left({\beta }_{1}^{2}+{\beta }_{2}^{2}\right)}^{2}{\left({\mu }_{1}^{2}+{\mu }_{2}^{2}\right)}^{2}}{\left({\sigma }_{{\theta }_{0}}^{2}+{\overline{\theta }}_{0}^{2}\right)}^{2}\\ & \quad >{\left(\left({\mu }_{1}^{2}+{\mu }_{2}^{2}\right){\overline{\theta }}_{0}^{2}+\frac{2}{{\beta }_{1}^{2}+{\beta }_{2}^{2}}\right)}^{2}-{\left(\frac{2}{{\beta }_{1}^{2}+{\beta }_{2}^{2}}\right)}^{2},\end{aligned}$$

or

$$\begin{aligned}&{\left(\left({\sigma }_{{\theta }_{0}}^{2}+{\overline{\theta }}_{0}^{2}\right)\frac{{\mu }_{1}^{6}-{\mu }_{2}^{6}}{{\mu }_{1}^{4}-{\mu }_{2}^{4}}+\frac{2}{{\beta }_{1}^{2}+{\beta }_{2}^{2}}\right)}^{2}-\frac{4{\gamma }_{1}^{2}{\gamma }_{2}^{2}{\mu }_{1}^{4}{\mu }_{2}^{4}}{{\left({\beta }_{1}^{2}+{\beta }_{2}^{2}\right)}^{2}{\left({\mu }_{1}^{2}+{\mu }_{2}^{2}\right)}^{2}}{\left({\sigma }_{{\theta }_{0}}^{2}+{\overline{\theta }}_{0}^{2}\right)}^{2}\\ & \quad >{\left(\left({\mu }_{1}^{2}+{\mu }_{2}^{2}\right){\overline{\theta }}_{0}^{2}+\frac{2}{{\beta }_{1}^{2}+{\beta }_{2}^{2}}\right)}^{2}.\end{aligned}$$
(33)

We also find that condition is equivalent to

$${\left(\left({\sigma }_{{\theta }_{0}}^{2}+{\overline{\theta }}_{0}^{2}\right)\frac{{\mu }_{1}^{6}-{\mu }_{2}^{6}}{{\mu }_{1}^{4}-{\mu }_{2}^{4}}+\frac{2}{{\beta }_{1}^{2}+{\beta }_{2}^{2}}\right)}^{2}>{\left(\left({\mu }_{1}^{2}+{\mu }_{2}^{2}\right){\overline{\theta }}_{0}^{2}+\frac{2}{{\beta }_{1}^{2}+{\beta }_{2}^{2}}\right)}^{2}.$$

Hence, if holds, then holds. That is, if \({W}_{l}^{Q}>{W}_{c}^{Q}\), then \({W}_{l}^{S}>{W}_{c}^{S}\). This is equivalent to: if \({W}_{l}^{S}<{W}_{c}^{S}\), then \({W}_{l}^{Q}<{W}_{c}^{Q}\).

Rights and permissions

Springer Nature or its licensor (e.g. a society or other partner) holds exclusive rights to this article under a publishing agreement with the author(s) or other rightsholder(s); author self-archiving of the accepted manuscript version of this article is solely governed by the terms of such publishing agreement and applicable law.

Reprints and permissions

About this article

Check for updates. Verify currency and authenticity via CrossMark

Cite this article

Wang, S. Incomplete contracts with disparity, uncertainty, information and incentives. Theory Decis (2024). https://doi.org/10.1007/s11238-024-09976-w

Download citation

  • Accepted:

  • Published:

  • DOI: https://doi.org/10.1007/s11238-024-09976-w

Keywords

Navigation